Re: Bijections (was OM = SIGMA1)

2007-11-21 Thread Bruno Marchal


Le 20-nov.-07, à 17:59, meekerdb a écrit :


 Bruno Marchal wrote:
 .

 But infinite ordinals can be different, and still have the same
 cardinality. I have given examples: You can put an infinity of linear
 well founded order on the set N = {0, 1, 2, 3, ...}.

 What is the definition of linear well founded order?  I'm familiar
 with well ordered, but how is linear applied to sets?  Just 
 curious.


By linear, I was just meaning a non branching order. A tree can be well 
founded too, meaning all its branches have a length given by an 
ordinal.

Bruno


http://iridia.ulb.ac.be/~marchal/


--~--~-~--~~~---~--~~
You received this message because you are subscribed to the Google Groups 
Everything List group.
To post to this group, send email to [EMAIL PROTECTED]
To unsubscribe from this group, send email to [EMAIL PROTECTED]
For more options, visit this group at 
http://groups.google.com/group/everything-list?hl=en
-~--~~~~--~~--~--~---



Re: Cantor's Diagonal

2007-11-21 Thread Bruno Marchal


Le 20-nov.-07, à 17:47, David Nyman a écrit :


 On 20/11/2007, Bruno Marchal [EMAIL PROTECTED] wrote:

 David, are you still there? This is a key post, with respect to the
 Church Thesis thread.

 Sorry Bruno, do forgive me - we seem destined to be out of synch at
 the moment.  I'm afraid I'm too distracted this week to respond
 adequately - back on-line next week at the latest.


Take it easy. It is the main advantage of electronical conversation, 
unlike the phone, we can answer at the best moment. But I appreciate 
you tell me.

See you next week.

Bruno

PS I'm afraid I have not the time to comment the last posts today, 
except for elementary question perhaps, but I will have more time soon 
(probably or hopefully tomorrow).






 David

 Hi,

 David, are you still there? This is a key post, with respect to the
 Church Thesis thread.

 So let us see that indeed there is no bijection between N and 2^N =
 2X2X2X2X2X2X... = {0,1}X{0,1}X{0,1}X{0,1}X... = the set of infinite
 binary sequences.

 Suppose that there is a bijection between N and the set of infinite
 binary sequences. Well, it will look like that, where again 
 represents the ropes:

 0  -- (1, 0, 0, 1, 1, 1, 0 ...
 1 --  (0, 0, 0, 1, 1, 0, 1 ...
 2 --- (0, 1, 0, 1, 0, 1, 1, ...
 3 --- (1, 1, 1, 1, 1, 1, 1, ...
 4 --- (0, 0, 1, 0, 0, 1, 1, ...
 5 (0, 0, 0, 1, 1, 0, 1, ...
 ...

 My sheep are the natural numbers, and my neighbor's sheep are the
 infinite binary sequences (the function from N to 2, the elements of
 the infinite cartesian product 2X2X2X2X2X2X... ).
 My flock of sheep is the *set* of natural numbers, and my neighbor's
 flock of sheep is the *set* of all infinite binary sequences.

 Now, if this:

 0  -- (1, 0, 0, 1, 1, 1, 0 ...
 1 --  (0, 0, 0, 1, 1, 0, 1 ...
 2 --- (0, 1, 0, 1, 0, 1, 1, ...
 3 --- (1, 1, 1, 1, 1, 1, 1, ...
 4 --- (0, 0, 1, 0, 0, 1, 1, ...
 5 (0, 0, 0, 1, 1, 0, 1, ...
 ...

 is really a bijection, it means that all the numbers 1 and 0 appearing
 on the right are well determined (perhaps in Platonia, or in God's
 mind, ...).

 But then the diagonal sequence, going from the left up to right down,
 and build from the list of binary sequences above:

 1 0 0 1 0 0 ...

 is also completely well determined (in Platonia or in the mind of a
 God).

 But then the complementary sequence (with the 0 and 1 permuted) is 
 also
 well defined, in Platonia or in the mind of God(s)

 0 1 1 0 1 1 ...

 But this infinite sequence cannot be in the list, above. The God in
 question has to ackonwledge that.
 The complementary sequence is clearly different
 -from the 0th sequence (1, 0, 0, 1, 1, 1, 0 ..., because it differs at
 the first (better the 0th)  entry.
 -from the 1th sequence (0, 0, 0, 1, 1, 0, 1 ...  because it differs at
 the second (better the 1th)  entry.
 -from the 2th sequence (0, 0, 0, 1, 1, 0, 1 ...  because it differs at
 the third (better the 2th)  entry.
 and so one.
 So, we see that as far as we consider the bijection above well
 determined (by God, for example), then we can say to that God that the
 bijection misses one of the neighbor sheep, indeed the sheep
 constituted by the infinite binary sequence complementary to the
 diagonal sequence cannot be in the list, and that sequence is also 
 well
 determined (given that the whole table is).

 But this means that this bijection fails. Now the reasoning did not
 depend at all on the choice of any particular bijection-candidate.  
 Any
 conceivable bijection will lead to a well determined infinite table of
 binary numbers. And this will determine the diagonal sequence and then
 the complementary diagonal sequence, and this one cannot be in the
 list, because it contradicts all sequences in the list when they cross
 the diagonal (do the drawing on paper).

 Conclusion: 2^N, the set of infinite binary sequences, is not
 enumerable.

 All right?

   Next I will do again that proof, but with notations instead of
 drawing, and I will show more explicitly how the contradiction arise.


 Exercice-training: show similarly that N^N, the set of functions from 
 N
 to N, is not enumerable.

 Bruno

 http://iridia.ulb.ac.be/~marchal/




 

http://iridia.ulb.ac.be/~marchal/


--~--~-~--~~~---~--~~
You received this message because you are subscribed to the Google Groups 
Everything List group.
To post to this group, send email to [EMAIL PROTECTED]
To unsubscribe from this group, send email to [EMAIL PROTECTED]
For more options, visit this group at 
http://groups.google.com/group/everything-list?hl=en
-~--~~~~--~~--~--~---



Re: Cantor's Diagonal

2007-11-21 Thread Bruno Marchal


Le 20-nov.-07, à 23:39, Barry Brent wrote :


 You're saying that, just because you can *write down* the missing
 sequence (at the beginning, middle or anywhere else in the list), it
 follows that there *is* no missing sequence.  Looks pretty wrong to me.

   Cantor's proof disqualifies any candidate enumeration.  You respond
 by saying, well, here's another candidate!  But Cantor's procedure
 disqualified *any*, repeat *any* candidate enumeration.

 Barry Brent


Torgny, I do agree with Barry. Any bijection leads to a contradiction, 
even in some effective way, and that is enough (for a classical 
logician).
But look what you write:

 On Nov 20, 2007, at 11:42 AM, Torgny Tholerus wrote:



 An ultrafinitist comment to this:
 ==
 You can add this complementary sequence to the end of the list.
 That will make you have a list with this complementary sequence
 included.

 But then you can make a new complementary sequence, that is not
 inluded.  But you can then add this new sequence to the end of the
 extended list, and then you have a bijection with this new sequence
 also.  And if you try to make another new sequence, I will add that
 sequence too, and this I will do an infinite number of times.


How could an ultrafinitist refute an argument by saying ... and this I 
will do an infinite number of times. ?




 So
 you will not be able to prove that there is no bijection...


Actually no. If you do what you described omega times, you will just 
end up with a set which can still be put in 1-1 correspondence with N 
(as shown in preceding posts on bijections)
To refute Cantor, here, you should do what you described a very big 
infinity of times, indeed an non enumerable infinity of times. But then 
you have to assume the existence of a non enumerable set at the start. 
OK?

Bruno
http://iridia.ulb.ac.be/~marchal/




 ==
 What is wrong with this conclusion?

 -- 
 Torgny



 Dr. Barry Brent
 [EMAIL PROTECTED]
 http://home.earthlink.net/~barryb0/




 



--~--~-~--~~~---~--~~
You received this message because you are subscribed to the Google Groups 
Everything List group.
To post to this group, send email to [EMAIL PROTECTED]
To unsubscribe from this group, send email to [EMAIL PROTECTED]
For more options, visit this group at 
http://groups.google.com/group/everything-list?hl=en
-~--~~~~--~~--~--~---



Re: Cantor's Diagonal

2007-11-21 Thread Bruno Marchal

Le 21-nov.-07, à 08:49, Torgny Tholerus a écrit :




  meekerdb skrev:Torgny Tholerus wrote:


 An ultrafinitist comment to this:
 ==
 You can add this complementary sequence to the end of the list.  That
 will make you have a list with this complementary sequence included.

 But then you can make a new complementary sequence, that is not
 inluded.  But you can then add this new sequence to the end of the
 extended list, and then you have a bijection with this new sequence
 also.  And if you try to make another new sequence, I will add that
 sequence too, and this I will do an infinite number of times.  So you
 will not be able to prove that there is no bijection...
 ==
 What is wrong with this conclusion?

 You'd have to insert the new sequence in the beginning, as there is no
 end of the list.



  Why can't you add something to the end of the list?  In an earlier 
 message Bruno wrote:

  Now omega+1 is the set of all ordinal strictly lesser than omega+1, 
 with the convention above. This gives {0, 1, 2, 3, ... omega} = {0, 1, 
 2, 3, 4, {0, 1, 2, 3, 4, }}.

  In this sentence he added omega to the end of the list of natural 
 numbers...




Adding something to the end or to the middle or to the beginning of an 
infinite  list, does not change the cardinality of that list. And in 
Cantor proof, we are interested only in the cardinality notion.

Adding something to the beginning or to the end of a infinite ORDERED 
list, well, it does not change the cardinal of the set involved, but it 
obviosuly produce different order on those sets, and this can give 
different ordinal, which denote type of order (isomorphic order).

The ordered set {0, 1, 2, 3, ...} has the same cardinality that the 
ordered set {1, 2, 3, 4, ... 0} (where by definition 0 is bigger than 
all natural numbers). But they both denote different ordinal, omega, 
and omega+1 respectively. Note that {1, 0, 2, 3, 4, ...} is a different 
order than {0, 1, 2, 3, ...}, but both order here are isomorphic, and 
correspond to the same ordinal (omega).

That is why 1+omega = omega, and omega+1 is different from omega. 
Adding one object in front of a list does not change the type of the 
order. Adding an element at the end of an infinite list does change the 
type of the order. {0, 1, 2, 3, ...} has no bigger element, but {1, 2, 
3, ... 0} has a bigger element. So, you cannot by simple relabelling of 
the elements get the same type of order (and thus they correspond to 
different ordinals).

OK?  (this stuff will not be used for Church Thesis, unless we go very 
far ...later).


Bruno


http://iridia.ulb.ac.be/~marchal/

--~--~-~--~~~---~--~~
You received this message because you are subscribed to the Google Groups 
Everything List group.
To post to this group, send email to [EMAIL PROTECTED]
To unsubscribe from this group, send email to [EMAIL PROTECTED]
For more options, visit this group at 
http://groups.google.com/group/everything-list?hl=en
-~--~~~~--~~--~--~---



Re: Cantor's Diagonal

2007-11-21 Thread Torgny Tholerus





Bruno Marchal skrev:

  
Le 20-nov.-07,  23:39, Barry Brent wrote :

  
  
You're saying that, just because you can *write down* the missing
sequence (at the beginning, middle or anywhere else in the list), it
follows that there *is* no missing sequence.  Looks pretty wrong to me.

  Cantor's proof disqualifies any candidate enumeration.  You respond
by saying, "well, here's another candidate!"  But Cantor's procedure
disqualified *any*, repeat *any* candidate enumeration.

Barry Brent

  
  

Torgny, I do agree with Barry. Any bijection leads to a contradiction, 
even in some effective way, and that is enough (for a classical 
logician).
  


What do you think of this "proof"?:

Let us have the bijection:

0  {0,0,0,0,0,0,0,...}
1  {1,0,0,0,0,0,0,...}
2  {0,1,0,0,0,0,0,...}
3  {1,1,0,0,0,0,0,...}
4  {0,0,1,0,0,0,0,...}
5  {1,0,1,0,0,0,0,...}
6  {0,1,1,0,0,0,0,...}
7  {1,1,1,0,0,0,0,...}
8  {0,0,0,1,0,0,0,...}
...
omega --- {1,1,1,1,1,1,1,...}

What do we get if we apply Cantor's Diagonal to this?

-- 
Torgny

--~--~-~--~~~---~--~~
You received this message because you are subscribed to the Google Groups Everything List group.  To post to this group, send email to [EMAIL PROTECTED]  To unsubscribe from this group, send email to [EMAIL PROTECTED]  For more options, visit this group at http://groups.google.com/group/everything-list?hl=en  -~--~~~~--~~--~--~---






Theory of Everything based on E8 by Garrett Lisi

2007-11-21 Thread George Levy
A theory of everyting is sweeping the Physics community.


The theory by Garrett Lisi is explained in this Wiki entry. 
http://en.wikipedia.org/wiki/An_Exceptionally_Simple_Theory_of_Everything


A simulation of E8 can be found a the New Scientist. 
http://www.newscientist.com/channel/fundamentals/dn12891-is-mathematical-pattern-the-theory-of-everything.html


The Wiki entry http://en.wikipedia.org/wiki/E8_%28mathematics%29 on E8 
is also interesting.


George

--~--~-~--~~~---~--~~
You received this message because you are subscribed to the Google Groups 
Everything List group.
To post to this group, send email to [EMAIL PROTECTED]
To unsubscribe from this group, send email to [EMAIL PROTECTED]
For more options, visit this group at 
http://groups.google.com/group/everything-list?hl=en
-~--~~~~--~~--~--~---



elaboration Re: Cantor's Diagonal

2007-11-21 Thread Barry Brent

The reason it isn't a bijection (of a denumerable set with the set of  
binary sequences):  the  pre-image (the left side of your map) isn't  
a set--you've imposed an ordering.  Sets, qua sets, don't have  
orderings.  Orderings are extra.  (I'm not a specialist on this stuff  
but I think Bruno, for example, will back me up.)  It must be the  
case that you won't let us identify the left side, for example, with  
{omega, 0, 1, 2, ... }, will you? For if you did, it would fall under  
Cantor's argument.

Barry

On Nov 21, 2007, at 10:33 AM, Torgny Tholerus wrote:

 Bruno Marchal skrev:
 Le 20-nov.-07, à 23:39, Barry Brent wrote :
 You're saying that, just because you can *write down* the missing  
 sequence (at the beginning, middle or anywhere else in the list),  
 it follows that there *is* no missing sequence. Looks pretty  
 wrong to me. Cantor's proof disqualifies any candidate  
 enumeration. You respond by saying, well, here's another  
 candidate! But Cantor's procedure disqualified *any*, repeat  
 *any* candidate enumeration. Barry Brent
 Torgny, I do agree with Barry. Any bijection leads to a  
 contradiction, even in some effective way, and that is enough (for  
 a classical logician).

 What do you think of this proof?:

 Let us have the bijection:

 0  {0,0,0,0,0,0,0,...}
 1  {1,0,0,0,0,0,0,...}
 2  {0,1,0,0,0,0,0,...}
 3  {1,1,0,0,0,0,0,...}
 4  {0,0,1,0,0,0,0,...}
 5  {1,0,1,0,0,0,0,...}
 6  {0,1,1,0,0,0,0,...}
 7  {1,1,1,0,0,0,0,...}
 8  {0,0,0,1,0,0,0,...}
 ...
 omega --- {1,1,1,1,1,1,1,...}

 What do we get if we apply Cantor's Diagonal to this?

 -- 
 Torgny

 

Dr. Barry Brent
[EMAIL PROTECTED]
http://home.earthlink.net/~barryb0/




--~--~-~--~~~---~--~~
You received this message because you are subscribed to the Google Groups 
Everything List group.
To post to this group, send email to [EMAIL PROTECTED]
To unsubscribe from this group, send email to [EMAIL PROTECTED]
For more options, visit this group at 
http://groups.google.com/group/everything-list?hl=en
-~--~~~~--~~--~--~---